PT69.S1.Q19 - the goblin fern, which requires

LivePumpkinLivePumpkin Free Trial Member
edited November 2017 in General 270 karma

Hi everyone!

I am having trouble with this question. I can see why the other 4 ACs are wrong but I am having trouble understanding why E is the correct answer. When negated, is E essentially saying that L comes AFTER the leaf litter becomes thin, which weakens the argument since it claims that L causes the thinning?

Thank you!

https://7sage.com/lsat_explanations/lsat-69-section-1-question-19/

Comments

Sign In or Register to comment.